Difference between revisions of "2002 AMC 10B Problems/Problem 25"
(→Solution) |
(→Solution) |
||
Line 5: | Line 5: | ||
== Solution == | == Solution == | ||
− | Let <math>x</math> be the sum of the | + | Let <math>x</math> be the sum of the integers and <math>y</math> be the number of elements in the list. Then we get the equations <math>\frac{x+15}{y+1}=\frac{x}{y}+2</math> and <math>\frac{x+15+1}{y+1+1}=\frac{x+16}{y+2}=\frac{x}{y}+2-1=\frac{x}{y}+1</math>. With a little work, the solution is found to be <math>y= \boxed{\textbf{(A)}\ 4} </math>. |
Revision as of 02:18, 29 January 2011
Problem
When 15 is appended to a list of integers, the mean is increased by 2. When 1 is appended to the enlarged list, the mean of the enlarged list is decreased by 1. How many integers were in the original list?
Solution
Let be the sum of the integers and be the number of elements in the list. Then we get the equations and . With a little work, the solution is found to be .